0 Daumen
204 Aufrufe

Aufgabe:


Sei \( A:=\sum \limits_{n=0}^{\infty} 2^{n}=1+2+4+8+\cdots \). Offenbar ist
\( \begin{aligned} A & =\sum \limits_{n=0}^{\infty} 2^{n}=\sum \limits_{n=1}^{\infty} 2^{n-1}=\sum \limits_{n=1}^{\infty} 2^{n} \cdot \frac{1}{2} \\ & =\frac{1}{2} \cdot \sum \limits_{n=1}^{\infty} 2^{n}=\frac{1}{2} \cdot\left(-1+\sum \limits_{n=0}^{\infty} 2^{n}\right)=\frac{1}{2} \cdot(-1+A), \end{aligned} \)
also \( A=\frac{1}{2}(A-1) \). Auflösen nach \( A \) liefert \( A=-1 \). Also gilt
\( 1+2+4+8+16+32+\cdots=-1 \)



Aufgabe: Wo ist der Fehler in obiger Rechnung? (Dazu gehört, dass Sie den genauen Ort
des Fehlers und die Art des Fehlers aufzeigen; sowie gegebenenfalls von weiteren Fehlern
in der Rechnung.)

Problem/Ansatz:

Hallo, habe ein wenig Probleme mit der obigen Aufgabe. Offensichtlich divergiert die geometrische Reihe.

Allerdings finde ich den Fehler einfach nicht, auch mit Einsetzen von Zahlen kann ich keinen offensichtlichen Fehler in den Umformungen erkennen.


Bin für jede Hilfe dankbar.

Avatar von

Wann darfst du ein Skalar vor die Reihe ziehen?

Stimmt, das Skalar darf nicht vor die Reihe gezogen werden, weil die Reihe divergiert, kann das sein?

1 Antwort

0 Daumen
 
Beste Antwort

Ich antworte mal mit einer Frage.

Wo liegt hier der Fehler?

\(2\infty = \infty \Rightarrow \infty = 0\) oder auch \(2=1\)

:-)

Avatar von 10 k

Ein anderes Problem?

Stell deine Frage

Ähnliche Fragen

Willkommen bei der Mathelounge! Stell deine Frage einfach und kostenlos

x
Made by a lovely community